JB - Airway and Breathing

अब Quizwiz के साथ अपने होमवर्क और परीक्षाओं को एस करें!

Which of the following assessment findings indicates a patent airway? A) Diaphoresis and a forceful cough B) Gurgling respirations and cyanosis C) Semiconscious with snoring respirations D) Loud inspiratory stridor and severe pallor

A A patent airway is one that is free of obstructions and secretions. Gurgling respirations indicate fluid in the airway. Snoring respirations indicate partial obstruction by the tongue, the most common cause of airway obstruction in unresponsive patients. Inspiratory stridor indicates upper airway swelling. Cyanosis, although a clear sign of hypoxia, is a relatively later finding. A patient who is coughing forcefully has a patent airway.

When providing positive-pressure ventilation (PPV) to a patient, it is important to remember that: A) PPV can impair venous return and cause hypotension. B) uncontrolled PPV often causes significant hypertension. C) bradycardia is commonly associated with any form of PPV. D) PPV draws blood back to the heart and improves cardiac output.

A During negative-pressure ventilation, the act of normal breathing, air is drawn into the lungs when intrathoracic decreases; this draws blood back to the heart and maintains cardiac output. Positive-pressure ventilation (PPV), the act of forcing air into the lungs, increases intrathoracic pressure and can impair venous return to the heart (preload), resulting in decreased cardiac output and hypotension. For this reason, it is important to ventilate the patient with just enough volume to produce visible chest rise and to avoid hyperventilating the patient. Bradycardia and hypertension are not commonly associated with PPV.

You are assessing an unresponsive man's respirations and note that he is taking irregular breaths that vary in volume and rate with periods of apnea. This breathing pattern is MOST consistent with: A) Biot's respirations. B) agonal respirations. C) Kussmaul's respirations. D) Cheyne-Stokes respirations.

A A pattern of irregular breathing that varies in depth and rate with sudden periods of apnea is referred to as Biot's respirations, also known as ataxic breathing. This respiratory pattern is commonly seen in patients with increased intracranial pressure, either from closed head trauma or hemorrhagic stroke. Clearly, patients displaying this respiratory pattern need ventilatory assistance. Agonal respirations, also called agonal gasps, are slow and irregular and produce minimal to no tidal volume; they are commonly observed shortly after a patient develops cardiac arrest. Kussmaul respirations are characterized by a pattern of deep and rapid breathing and are commonly observed in patients with diabetic ketoacidosis. Cheyne-Stokes respirations follow a crescendo-decrescendo pattern in which the patient breaths fast, then slow, and has intermittent periods of apnea.

An adult patient without respiratory distress, who is breathing at a rate of 14 times per minute and has adequate tidal volume, will MOST likely: A) eliminate adequate carbon dioxide from the body. B) maintain a constant oxygen saturation of 100%. C) have an arterial PO2 of 60 and 80 torr (mm Hg). D) have adventitious breath sounds upon auscultation.

A Adequate breathing in the adult is characterized by, among other signs, a respiratory rate between 12 and 20 breaths/min, adequate tidal volume (depth of breathing), and a regular pattern of inhalation and exhalation. An adequately breathing patient will be able to eliminate adequate amounts of carbon dioxide from the body, maintain an oxygen saturation (SpO2) of greater than 95%, and will likely have a normal arterial PO2 of 80 to 100 torr (mm Hg). Adventitious breath sounds are abnormal breath sounds (ie, wheezing, rales, rhonchi), and may indicate inadequate ventilation.

You are en route to the hospital with a patient in respiratory extremis. You have administered a sedative and a paralytic drug to the patient, but have been unable to successfully intubate him after two attempts. Your EMT partner tells you that the patient's oxygen saturation is 98% and that his heart rate is 70 beats/min. Your estimated time of arrival at the hospital is 7 minutes. Your MOST appropriate next action should be to: A) resume bag-mask ventilations at 12 breaths/min and insert a multilumen or a supraglottic airway device. B) preoxygenate the patient for approximately 30 seconds and reattempt to successfully intubate the patient. C) ask your partner to resume bag-mask ventilations as you prepare to perform a surgical cricothyrotomy. D) insert a Combitube to secure the patient's airway and ventilate him at a rate of 20 to 24 breaths/min.

A Although your intubation attempts have been unsuccessful, the patient's heart rate and oxygen saturation are stable. If endotracheal intubation is unsuccessful, you must always resume ventilations with a bag-mask device and 100% oxygen; insert an oral airway to help maintain patency of the airway. Because you are only 7 minutes away from the hospital, it would be quicker to insert a multilumen (eg, Combitube) or supraglottic (eg, King LT, CobraPLA, LMA) airway device, after reoxygenating him with a bag-mask device and 100% oxygen. Do NOT hyperventilate the patient; ventilate him at a rate of 10 to 12 breaths/min. If you cannot adequately ventilate the patient with a bag-mask device and are unable to insert a multilumen or supraglottic airway device, you should proceed with a cricothyrotomy.

When assessing an unresponsive patient, you note that he is not breathing. Which of the following airway devices or interventions is contraindicated? A) Nasotracheal intubation B) Esophageal Combitube C) Orotracheal intubation D) Laryngeal mask airway

A Because the endotracheal tube is advanced when the patient inhales (the vocal cords are open at their widest during inhalation), blind nasotracheal intubation is contraindicated in apneic patients. Orotracheal intubation, as well as alternative airway devices (ie, Combitube, laryngeal mask airway [LMA], King LT airway), would be appropriate devices used to secure the patient's airway, provided that he does not have an intact gag reflex.

A 49-year-old man presents with acute shortness of breath. He is conscious, but confused, and is gasping for air. The pulse oximeter reads 79% on room air. Initial management should consist of: A) some form of ventilatory assistance. B) 100% oxygen via nonrebreathing mask. C) sedation and endotracheal intubation. D) insertion of an oropharyngeal airway.

A Confusion (indicates decreased cerebral perfusion), gasping for air (air hunger), and an oxygen saturation of 79% are clear signs that this patient has inadequate ventilation; therefore, he will require some form of ventilatory assistance, such as a bag-mask device with supplemental oxygen. The fact that he is awake negates the use of an oral airway. Should the patient deteriorate further, endotracheal intubation should be considered.

A 60-year-old man reports dyspnea. While auscultating his chest, you hear fine, moist, thin sounds in all lung fields. What is this MOST suggestive of? A) Fluid in the small lower airways B) Fluid in the large lower airways C) Mucous plugs in the alveoli D) Mild to moderate bronchospasm

A Crackles (formerly called rales), which are fine, moist, thin sounds that are difficult to auscultate, represent fluid in the small lower airways and are indicative of early pulmonary edema (ie, congestive heart failure). Rhonchi are loud rattling sounds that can often be heard without a stethoscope and are indicative of fluid in the larger airways (ie, severe pulmonary edema). Bronchospasm typically presents with wheezing, not crackles or rhonchi.

You are ventilating an intubated patient and note decreased compliance with each delivered ventilation. Which of the following conditions would be the LEAST likely cause of this? A) Beta-2 receptor stimulation B) Diffuse bronchospasm C) Tension pneumothorax D) Surfactant deficiency

A Decreased ventilatory compliance (increased ventilatory resistance) is the result of any condition that causes a barrier to airflow into the lungs. Such conditions include bronchospasm, tension pneumothorax, upper or lower airway obstruction, and surfactant deficiency. Surfactant is a protein substance that lines the alveolar walls and decreases alveolar surface tension. Stimulation of beta-2 receptors causes bronchodilation, which would facilitate - not impede - airflow into the lungs.

What is the approximate minute alveolar volume of a patient who is breathing at a rate of 26 breaths/min with an estimated tidal volume of 450 mL? A) 8,200 mL B) 9,400 mL C) 10,100 mL D) 11,700 mL

A Minute alveolar volume, also called minute alveolar ventilation, is the volume of air that reaches the alveoli and participates in pulmonary gas exchange each minute. It is computed by multiplying the patient's tidal volume - less the dead space volume - and the patient's respiratory rate. Dead space volume - the volume of air that lingers in the upper airway and does not reach the alveoli - is approximately 30% (or 1 mL per pound of body weight) of the patient's tidal volume. Thus, if the patient's tidal volume is 450 mL, the actual volume of air that enters the lungs per breath is approximately 315 mL (450 mL - 135 mL [30% of 450 mL] = 315 mL). Therefore, the patient's minute alveolar volume is approximately 8,200 mL (315 mL x 26 breaths/min = 8,190 mL).

Which of the following interventions is MOST appropriate when treating an unresponsive adult with a severe foreign body airway obstruction? A) Chest compressions, laryngoscopy and use of Magill forceps, and cricothyrotomy B) Abdominal thrusts, finger sweeps of the mouth, and translaryngeal jet ventilation C) Subdiaphragmatic thrusts, oropharyngeal visualization, and Combitube insertion D) Back slaps, visualization of the oropharynx, finger sweeps, and chest compressions

A Patients with a severe (complete) foreign body airway obstruction (FBAO) are unable to move any air; therefore, rapid intervention is crucial to their survival. In the unresponsive patient - regardless of age - initial management for a severe FBAO involves chest compressions, visualization of the mouth (finger sweeps are indicated only if you can see the object), and attempts to ventilate. If a few cycles of basic interventions are unsuccessful, you should proceed to advanced techniques; visualize the upper airway with a laryngoscope and remove the object with Magill forceps if you can see it. If this technique is unsuccessful, resume basic interventions and perform a surgical cricothyrotomy. Needle cricothyrotomy and translaryngeal jet ventilation is contraindicated in the presence of a severe upper airway obstruction because the jet ventilator does not allow for exhalation. Subdiaphragmatic (abdominal) thrusts are indicated for a responsive adult or child with a severe FBAO. Back slaps and chest thrusts are indicated for a responsive infant with a severe FBAO. Insertion of a Combitube or other esophageal airway device would be of no benefit; obturation of the esophagus will not help the patient whose trachea is obstructed.

Which of the following statements regarding physiologic dead space is correct? A) Diffuse alveolar collapse increases the size of the physiologic dead space. B) Physiologic dead space is normally found in the trachea and large bronchi. C) COPD patients typically have a marked decrease in physiologic dead space. D) Unlike anatomic dead space, the size of the physiologic dead space does not change.

A Physiologic dead space is any portion of the lower respiratory tract in which gas exchange does not occur because of abnormal processes such as alveolar collapse (atelectasis), alveolar damage, or fluid in the alveoli. Because COPD causes destruction of the inner lining of the alveoli, physiologic dead space increases in patients with the disease. Approximately 30% of a person's tidal volume remains in the anatomic dead space - the trachea and large bronchi - and never makes it to the lungs to participate in pulmonary respiration. Unlike the physiologic dead space, which increases in size in patients with respiratory diseases, the size of the anatomic dead space remains relatively constant.

Prior to applying a nonrebreathing mask on a responsive patient with respiratory distress, you should: A) ensure that the reservoir bag is fully inflated. B) set the oxygen flowmeter to no more than 10 L/min. C) ask the patient to exhale fully and then hold his breath. D) place the patient in a lateral recumbent position.

A Prior to placing a nonrebreathing mask on a patient, you must ensure that the reservoir bag is completely filled. If it is not prefilled, the nonrebreathing mask will not deliver high concentrations of oxygen. The appropriate oxygen flow rate for a nonrebreathing mask is 12 to 15 L/min. A responsive patient with adequate breathing and no trauma should be placed in a position of comfort. A recumbent position is necessary if the patient is unresponsive, is not injured, and is breathing adequately.

You have been ventilating an unresponsive apneic 42-year-old male for approximately 12 minutes. After securing his airway with an endotracheal tube and confirming proper ET tube placement, you should: A) insert an orogastric or nasogastric tube. B) deliver one breath every 6 to 8 seconds. C) provide a tidal volume of about 10 mL/kg. D) maintain an end-tidal CO2 of 45 to 50 mm Hg.

A Prolonged bag-mask ventilations often result in gastric distention. After the patient has been intubated, excessive air in the stomach may impede your ability to deliver adequate tidal volume. Therefore, you should consider inserting an oro- or nasogastric tube after the ET tube has been placed and proper positioning of the tube has been confirmed. When ventilating an apneic adult who has a pulse, deliver one breath every 5 to 6 seconds (10 to 12 breaths/min). A ventilation rate of 8 to 10 breaths/min (one breath every 6 to 8 seconds) is appropriate for patients in cardiac arrest, after an advanced airway device has been placed. Normal tidal volume for the average adult male is 5 to 7 mL/kg (about 500 mL); a tidal volume of 10 mL/kg is clearly too much volume and may result in barotrauma. Remember to ventilate the patient over 1 second - just enough to produce visible chest rise. Capnography should be used in conjunction with intubation to quantify adequate CO2 elimination. In the apneic patient with a pulse, you should maintain an end-tidal CO2 (ETCO2) between 35 and 45 mm Hg. An ETCO2 reading greater than 45 mm Hg in an intubated apneic patient with a pulse usually indicates that you are ventilating too slowly.

In which of the following conditions would you MOST likely detect a drop in systolic blood pressure during inhalation? A) Severe asthma B) Chronic bronchitis C) Pulmonary edema D) Simple pneumothorax

A Pulsus paradoxus, a drop in the systolic blood pressure of 10 mm Hg or more during inhalation, indicates physical restriction of cardiac movement during inhalation. Pulsus paradoxus can also be characterized by a marked weakening (or even disappearance) of the pulse during inhalation. Pulsus paradoxus may be observed in patients experiencing a severe asthma attack or an exacerbation of emphysema, in which case air-trapping in the lungs leads to pulmonary hyperinflation. As the hyperinflated lungs put pressure against the heart during inhalation, cardiac movement can be physically restricted. Pulsus paradoxus may also be observed in patients with a severe pericardial tamponade.

Which of the following breath sounds is also referred to as crackles? A) Rales B) Rhonchi C) Vesicular D) Wheezing

A Rales are commonly referred to as crackles. They are fine, moist, thin sounds that are often difficult to hear and indicate fluid in the smaller lower airways, such as what occurs during early pulmonary edema. Rhonchi is much louder than rales and produce a rattling sound; it indicates fluid in the larger lower airways and is often encountered in patients with severe pulmonary edema or bronchitis. Wheezing is a whistling sound that indicates bronchospasm; wheezes may be heard during inspiration, expiration, or both, and are most often encountered in patients with reactive airway diseases (ie, asthma, bronchiolitis). Vesicular breath sounds, which are normal, are characterized by inspiratory sounds that last longer than expiratory sounds; they are normally faint.

The exchange of oxygen and carbon dioxide between inspired air and the pulmonary capillaries is called: A) external respiration. B) internal respiration. C) pulmonary ventilation. D) intrapulmonary shunting.

A Respiration is defined as the exchange of oxygen and carbon dioxide between the body and its environment. External (pulmonary) respiration is the exchange of oxygen and carbon dioxide between inspired air and the pulmonary capillaries. Internal (cellular) respiration is the transfer of oxygen and carbon dioxide between the capillaries and tissue cells. Pulmonary ventilation is defined as the movement of air into and out of the lungs. Intrapulmonary shunting is an abnormal process in which external respiration is impaired due to alveolar dysfunction or damage (eg, atelectasis, pulmonary edema). Deoxygenated blood from the right side of the heart bypasses the nonfunctional alveoli and returns to the left side of the heart, still deoxygenated. The more diffuse the alveolar dysfunction, the greater the degree of intrapulmonary shunting.

A selective beta-2 adrenergic agonist will produce which of the following effects? A) Bronchodilation B) Increased inotropy C) Increased vascular resistance D) Increased parasympathetic tone

A Selective beta-2 adrenergic agonists, such as albuterol (Proventil, Ventolin) and metaproterenol (Alupent), cause bronchial smooth muscle relaxation, resulting in bronchodilation. Therefore, their use is indicated for patients with reactive airway diseases and accompanying bronchospasm. Beta-1 agonists, such as epinephrine, affect the heart, resulting in increased contractility (inotropy), heart rate (chronotropy), and electrical conduction velocity (dromotropy). Alpha agonists, such as norepinephrine (Levophed), stimulate receptors in the vasculature, resulting in an increase in vascular resistance and arterial blood pressure. Beta agonists do not stimulate the parasympathetic nervous system.

In which of the following situations would it be MOST appropriate to insert a King LT supraglottic airway? A) Comatose patient who ingested aspirin B) Apneic patient with cancer of the esophagus C) Unresponsive patient with a minimal gag reflex D) Unresponsiveness following oven cleaner ingestion

A The King LT is a supraglottic airway device that can be used as an alternative to endotracheal intubation or if endotracheal intubation has been attempted without success. It has been shown to provide better ventilation than the bag-mask device. The King LT is advantageous in that laryngoscopy and visualization of the vocal cords are not required, and the insertion technique requires less training than endotracheal intubation. The King LT is only used in unresponsive patients; it is contraindicated in those with an intact gag reflex (even if the gag reflex is minimal), in patients who ingested a caustic substance (eg, oven cleaner [strong acid]), and in patients with known esophageal disease (eg, cancer, varices).

A patient was bitten by fire ants and is unresponsive. He has severe edema to the face and neck and generalized urticaria. Breath sounds are difficult to hear, and loud inspiratory stridor is noted. Which of the following interventions has the highest priority? A) Endotracheal intubation B) Administration of epinephrine C) Administration of diphenhydramine D) Immediate transport

A The patient's upper airway is rapidly swelling as evidenced by the markedly diminished breath sounds and loud inspiratory stridor. If the patient is not promptly intubated before the airway completely closes, you will have to perform a cricothyrotomy or the patient will die. After the airway is secured and the patient is being adequately oxygenated and ventilated, you should administer epinephrine, followed by diphendydramine (Benadryl). Transport the patient as soon as possible.

In an otherwise healthy individual, breathing is primarily stimulated by: A) an increase in arterial CO2. B) an increase in arterial O2. C) a decrease in arterial O2. D) a decrease in arterial CO2.

A The primary stimulus to breathe in an otherwise healthy individual is an increase in the level of arterial carbon dioxide and a decrease in the pH of the cerebrospinal fluid (CSF). A decreased level of arterial oxygen is also a powerful stimulus to breathe (hypoxic drive), but is not the primary stimulus in otherwise healthy individuals.

A 56-year-old woman presents with acute respiratory distress. She is confused, has cyanosis around her mouth, and can only speak in two-word sentences. You should: A) assist her ventilations with a bag-mask device. B) give high-flow oxygen via nonrebreathing mask. C) perform immediate nasotracheal intubation. D) administer a beta-2 agonist via nebulizer.

A This patient is not ventilating adequately, as evidenced by the cyanosis around her mouth (perioral cyanosis), her state of confusion (indicates cerebral hypoxia), and her inability to speak more than two words at a time (two-word dyspnea). She needs some form of positive-pressure ventilation assistance, such as what is provided with a bag-mask device. Passive oxygenation (ie, nonrebreathing mask) is of little or no benefit if the patient is not ventilating adequately. Nasotracheal intubation can be considered, but not before restoring minute volume with less invasive means. Further assessment will determine the need for a nebulized beta-2 agonist.

When arterial oxygen levels in the body fall, chemoreceptors in the brain send messages to the diaphragm via the: A) phrenic nerve. B) vagus nerve. C) brainstem. D) medulla.

A When low oxygen levels are detected by the chemoreceptors in the blood, messages are sent to the diaphragm via the phrenic nerve, which originates in between the third and fifth cervical vertebrae. Injury to the cervical spine in this area can sever the phrenic nerve and result in respiratory paralysis. The chemoreceptors in the brain are located within the medulla oblongata, a part of the brainstem.

You are ventilating an unresponsive, apneic 50-year-old man. He has a pulse, but it is rapid and weak. When ventilating this patient, it is MOST important to: A) ventilate until the chest visibly rises. B) promptly insert an endotracheal tube. C) deliver one breath every 3 seconds. D) provide 8 to 10 breaths per minute.

A When ventilating any apneic patient, you should deliver each breath over a period of 1 second - just enough to produce visible chest rise. You should avoid hyperventilating the patient as this increases the incidence of gastric distention and the associated risks of regurgitation and aspiration; it may also impede venous return to the heart secondary to increased intrathoracic pressure. In the apneic adult with a pulse, deliver one breath every 5 to 6 seconds (10 to 12 breaths/min). If the patient (adult, child, or infant) is apneic and pulseless, ventilations should be provided at a rate of 8 to 10 breaths/min (one breath every 6 to 8 seconds) after an advanced airway device has been inserted. Endotracheal intubation is not always an immediate priority, especially if you are able to maintain a patient's airway with manual positioning and a simple adjunct and are able to adequately ventilate him or her with a bag-mask device.

A 56-year-old male presents with respiratory distress. He appears tired and is slow to answer your questions. He is taking a series of quick breaths, followed by prolonged exhalation. On the basis of these clinical findings, you should: A) provide some form of positive-pressure ventilation. B) administer a beta-2 agonist via a hand-held nebulizer. C) give oxygen via nonrebreathing mask and reassess him. D) intubate his trachea with the aid of pharmacologic agents.

A Your patient is not breathing adequately. Brief inhalations followed by prolonged exhalation will not produce adequate tidal volume; as a result, minute volume will decrease. His tired appearance and delay in answering your questions are signs of decreased cerebral perfusion secondary to inadequate ventilation. You must restore his tidal volume by providing some form of positive-pressure ventilatory assistance (eg, bag-mask ventilation). Patients with reduced tidal volume have inadequate negative-pressure ventilation and need positive-pressure ventilation. Passive oxygenation via nonrebreathing mask is appropriate for patients with signs of hypoxemia whose tidal volume is adequate (eg, they are not breathing shallowly). Bronchospasm may be the underlying cause of the patient's respiratory distress; however, because of his poor respiratory effort, he will not obtain the full benefit of a beta-2 agonist via hand-held nebulizer. A small-volume nebulizer connected inline with the bag-mask device would be more effective. If the patient's mental status does not improve despite adequately performed ventilation assistance, intubation should be considered. Although the patient is tired and is slow to answer your questions, he is still conscious. Therefore, pharmacologic agents (ie, sedation, neuromuscular blockade) will be required if it is determined that intubation is necessary.

A 44-year-old male was found unresponsive by his wife. According to the wife, he had been drinking bourbon whiskey all day. He is unresponsive; has slow, shallow respirations; and a slow, weak pulse. You should: A) promptly intubate his trachea and support his ventilations. B) insert an oral airway and give oxygen via nonrebreathing mask. C) ventilate him at a rate of 24 breaths/min with a bag-mask device. D) insert a laryngeal mask airway to prevent aspiration if he vomits.

A Your patient's airway is in immediate jeopardy! He is unresponsive, is unable to maintain his own airway, and has a stomach full of alcohol. Furthermore, the central nervous system depressant effects of alcohol are causing respiratory and cardiopulmonary insufficiency (eg, hypoventilation, bradycardia). First, you must protect his airway from aspiration; this is most effectively accomplished by intubating his trachea. Slow, shallow respirations do not provide adequate minute volume (his bradycardia is likely hypoxia-induced) and should be treated with positive-pressure ventilation; a nonrebreathing mask is not sufficient. Assist the patient's ventilations; provide 10 to 12 breaths/min with enough volume to produce visible chest rise. Hyperventilation - regardless of whether the patient is intubated - has been shown to be detrimental; it hyperinflates the lungs, which squeezes the heart and reduces preload and subsequent cardiac output. Although the laryngeal mask airway (LMA) has been shown to provide better positive-pressure ventilation than the bag-mask device, it does not eliminate the risk of aspiration.

You are transporting a woman with a history of COPD, who called EMS when her difficulty breathing suddenly worsened. She is receiving oxygen at 4 L/min via nasal cannula, is on a cardiac monitor, and has an IV line in place. During your reassessment, you note that she is responsive to pain only, is increasingly tachycardic, and is developing cyanosis around her mouth. You should: A) insert a Combitube to protect her airway. B) assist her breathing with a bag-mask device. C) apply a nonrebreathing mask at 15 L/min. D) place her on her side in case she vomits.

B A decreased level of consciousness, tachycardia, and cyanosis indicate that your patient is no longer breathing adequately and has worsened hypoxemia. You should begin assisting her breathing with a bag-mask device and 100% oxygen. Patients with inadequate breathing (ie, shallow breathing [reduced tidal volume], respirations that are too fast or too slow) need some form of positive-pressure ventilation to maintain adequate minute volume; this cannot be accomplished with a nonrebreathing mask. If the patient's level of consciousness deteriorates to a point of unresponsiveness, advanced airway management (ie, endotracheal intubation, Combitube, CobraPLA) should be considered. The recovery position is appropriate for unresponsive patients with adequate breathing; this patient is not breathing adequately.

During your assessment of a patient with labored breathing, you note asymmetric chest wall movement. This indicates that: A) one lung is inundated with fluid. B) airflow into one lung is reduced. C) bilateral bronchospasm is present. D) several ribs are fractured in several places.

B Asymmetric chest movement - when one side of the chest (hemithorax) moves less than the other - indicates that airflow into one lung is reduced. It could be the result of conditions such as a tension pneumothorax or excess fluid in one lung (ie, unilateral pleural effusion). Bilateral bronchospasm results in decreased air movement in both lungs, and would cause both sides of the chest to move minimally. When several ribs are fractured in more than one place, a free-floating segment of fractured ribs is created. This free-floating (flail) segment - not necessarily the entire hemithorax - collapses during inhalation and bulges during exhalation (paradoxical chest movement).

Which of the following is the LEAST reliable when assessing the oxygenation and/or perfusion status of an adult patient? A) Skin condition B) Capillary refill C) Mental status D) Pulse oximetry

B Because peripheral perfusion decreases with age, capillary refill time is a less reliable indicator of oxygenation and perfusion in adults; it is more reliable in children younger than 6 years of age. It is important to remember that factors such as cold temperatures can affect capillary refill time. Skin condition and mental status are excellent indicators of a person's oxygenation and perfusion status, regardless of age. Although not without limitations (ie, abnormal hemoglobin binding [CO toxicity]), pulse oximetry is a more reliable indicator of oxygenation than capillary refill time.

Normal breathing in a resting adult male: A) generally produces a tidal volume of about 8 to 10 mL/kg. B) should be marked by only subtle changes in rate or regularity. C) typically causes a weakening of the pulse during peak inspiration. D) is characterized by a short inspiratory time and prolonged expiratory time.

B Breathing should be effortless, with inspiratory and expiratory times being roughly equal. A brief (or normal) inspiratory time followed by a prolonged expiratory time is a sign of inadequate ventilation. The healthy adult male has a tidal volume of 5 to 7 mL/kg (about 500 mL). During normal breathing, subtle changes in rate and/or regularity are normal; grossly obvious changes, however, are not. A weakening of the pulse during peak inspiration - called pulsus paradoxus - is not normal, and is often associated with conditions such as severe asthma, COPD, and pericardial tamponade.

A 66-year-old male presents with labored breathing. He is conscious and alert; however, he is tachypneic, tachycardic, and is experiencing difficulty speaking. He has a history of hypertension and congestive heart failure. Auscultation of his lungs reveals diffuse coarse crackles and his oxygen saturation is 86%. Which of the following interventions will be of MOST benefit to this patient? A) Ventilation with a bag-mask device B) Continuous positive airway pressure C) High-flow oxygen via nonrebreathing mask D) Sedation, chemical paralysis, and intubation

B Continuous positive airway pressure (CPAP), a method of transmitting positive pressure into the lungs of a spontaneously breathing patient, is used in conjunction with positive-end expiratory pressure (PEEP), which is set between 2.5 and 10 cm H2O. CPAP causes the patient to exhale against positive pressure; this reexpands collapsed alveoli and forces fluid from the alveoli. CPAP is useful in treating patients with diffuse atelectasis and pulmonary edema; it reduces the work of breathing and improves pulmonary respiration. Most patients who are placed on CPAP are anxious initially; however, with an effective mask-to-face seal and reassurance, clinical improvement is often noted rather quickly. Signs of improvement include decreased work of breathing, increased ease in speaking, increasing Sp02, and decreases in heart rate and respiratory rate. When used in conjunction with medications to treat pulmonary edema (eg, nitroglycerin), CPAP has been shown to reduce the need for intubation. If CPAP fails, however, you must be able to recognize patient deterioration and be prepared to assist ventilations with a bag-mask device. CPAP is contraindicated in patients with slow, shallow (reduced tidal volume) breathing and in patients who are unable to follow simple commands; these patients require assisted ventilation with a bag-mask device and possibly intubation. High-flow (12 to 15 L/min) oxygen via nonrebreathing mask increases the fraction of inspired oxygen (FiO2); however, it is most effective if there are no barriers to diffusion in the lungs (ie, atelectasis, pulmonary edema).

Which of the following techniques or devices will provide the highest tidal volume to a patient? A) Nonrebreathing mask at 15 L/min B) Pocket face mask with oxygen attached C) One-person bag-mask device technique D) Continuous positive airway pressure (CPAP)

B Of the choices listed, the pocket face mask would provide the highest tidal volume. This is because the rescuer is breathing air from his/her own lungs into the patient's lungs and both of his/her hands are freed to maintain an effective mask-to-face seal. The bag-mask device can provide adequate tidal volume; however, greater tidal volumes are achieved when two rescuers are ventilating the patient. In the two-rescuer bag-mask technique, one rescuer maintains a mask seal while the other squeezes the bag. The nonrebreathing mask is a device that delivers oxygen passively; it does not deliver positive-pressure. Continuous positive airway pressure (CPAP) relies upon adequate tidal volume to be effective; the patient receives maximum benefit from CPAP during the exhalation phase, in which positive-pressure is directed to the lower airways, which forces fluid from the alveoli and opens the bronchioles.

Which of the following respiratory diseases is associated with decreased alveolar elasticity, air trapping in the lungs, and an increase in residual lung volume? A) Asthma B) Emphysema C) Bronchiectasis D) Chronic bronchitis

B Emphysema is characterized by destructive changes in the alveolar walls, including alveolar collapse and decreased alveolar elasticity. As a result, air becomes trapped in the lungs and residual lung volume increases; this explains why patients with emphysema have a barrel-shaped chest. Chronic bronchitis is caused by an increase in the number and size of mucus-producing cells (Goblet cells), resulting in chronic inflammation and excessive mucus production in the bronchial tree. Unlike emphysema, however, the alveoli generally are not seriously affected. Emphysema and chronic bronchitis are both chronic obstructive pulmonary diseases (COPD), and are most often the result of long-term cigarette smoking. Bronchiectasis is a disease that causes localized, irreversible dilation of part of the bronchial tree; it is caused by a pus-producing bacterial infection of the bronchial wall. In bronchiectasis, affected bronchi are easily collapsible, which results in impairment of airflow and clearance of pulmonary secretions. Because patients with chronic bronchitis experience frequent respiratory infections, bronchiectasis is especially common in this patient population. Asthma - a reactive airway disease - is a reversible condition characterized by contraction of the bronchiole smooth muscle (bronchospasm), inflammation of the bronchial walls, and mucus plugging, which results in impaired airflow through the bronchioles. Common triggers to asthma include stress, temperature changes, and respiratory irritants (eg, cigarette smoke). Asthma is also an obstructive lung disease; however, unlike emphysema and chronic bronchitis, it is episodic rather than chronic.

All of the following medications are used to "chemically paralyze" a patient to facilitate endotracheal intubation, EXCEPT: A) Pavulon. B) etomidate. C) Norcuron. D) rocuronium.

B Etomidate (Amidate) is a nonnarcotic, nonbarbiturate, sedative-hypnotic drug. It is not a neuromuscular blocking agent - that is, it does not induce paralysis. Sedative-hypnotic drugs are used to sedate the patient before administering a neuromuscular blocking agent (paralytic) and performing orotracheal intubation. Pancuronium bromide (Pavulon), vecuronium bromide (Norcuron), and rocuronium bromide (Zemuron) are all examples of neuromuscular blocking agents that induce chemical paralysis; they all function at the neuromuscular junction and relax skeletal (striated) muscle by impeding the action of acetylcholine.

Which of the following occurs when a patient is hypoventilating? A) Decreased PaCO2, increased PaO2, increased pH B) Increased PaCO2, decreased PaO2, decreased pH C) Decreased PaO2, decreased PaCO2, increased pH D) Increased PaO2, increased PaCO2, decreased pH

B Hypoventilation, by definition, is a condition in which the body does not eliminate adequate amounts of carbon dioxide. This is typically the result of inadequate ventilation (eg, reduced tidal volume, bradypnea, etc.). Because the hypoventilating patient is retaining carbon dioxide, the PaCO2 would increase and the pH of the blood and CSF would decrease (acidosis). Insufficient carbon dioxide elimination is accompanied by insufficient oxygen intake; therefore, the PaO2 would decrease.

Which of the following would MOST likely cause laryngeal spasm? A) Inhalation of cool, humidified mist B) Extubation of a semiconscious patient C) Insertion of a laryngeal mask airway D) Forceful positive-pressure ventilation

B Laryngeal spasm - spasmodic closure of the vocal cords - is often caused by an overly aggressive intubation technique. It may also occur during extubation, especially if the patient is semiconscious (one of the many reasons to avoid field extubation!). Cool, humidified mist is often used to reduce upper airway edema, such as that caused by croup. Insertion of the laryngeal mask airway (LMA) is not associated with a high incidence of laryngeal spasm because the device is not placed in between the vocal cords. The most significant complications of forceful positive-pressure ventilation include barotrauma, which may cause a pneumothorax; gastric distention, which may cause regurgitation and aspiration in the nonintubated patient; and reduced venous return to the heart (preload), which may cause hypotension.

Which of the following patients is the BEST candidate for nasotracheal intubation? A) Unresponsive, mandibular fracture, apnea B) Semiconscious, pulmonary edema, tachypnea C) Restless, audible wheezing, low oxygen saturation D) Cerebrospinal rhinorrhea, semiconscious, bradypnea

B Nasotracheal intubation is indicated for patients who are breathing spontaneously, but require definitive airway management to prevent further deterioration of their condition. Conscious patients or patients with an altered mental status and an intact gag reflex, who are in respiratory failure due to conditions such as COPD, asthma, or pulmonary edema, are typical candidates for nasotracheal intubation. Because the tracheal tube is advanced during inhalation - the point at which the glottis is open the widest - nasotracheal intubation is contraindicated in apneic patients. It is also contraindicated in patients with nasal trauma or evidence of a cribriform plate fracture (ie, cerebrospinal rhinorrhea). A restless patient with audible wheezing and a low oxygen saturation should be treated initially with supplemental oxygen and a beta-2 agonist drug (ie, albuterol) or a parasympathetic bronchodilator (ie, Atrovent). If the patient's condition is refractory to supplemental oxygen and several bronchodilator treatments, and he or she is still breathing spontaneously, nasotracheal intubation would be a consideration.

Which of the following is the MOST reliable immediate indicator that you have successfully placed an endotracheal (ET) tube into the trachea? A) Vapor mist is visible inside the ET tube during exhalation. B) Visualization of the ET tube passing in between the vocal cords. C) The presence of breath sounds that are clear and equal bilaterally. D) Paper color change in an end-tidal CO2 detector during exhalation.

B Of the choices listed, the most reliable immediate indicator that you have successfully intubated the trachea is to see the tip of the ET tube pass through the vocal cords with your own eyes. When you remove the laryngoscope blade from the patient's mouth, however, you must ensure continued correct placement of the ET tube. Methods and techniques used to accomplish this include quantitative waveform capnography (preferred), the presence of clear and equal breath sounds bilaterally, and an absence of epigastric sounds. The colorimetric ETCO2 detector provides qualitative data regarding correct ET tube placement and is fairly reliable, but not as reliable as waveform capnography. The esophageal detector device (EDD) is also an alternative option for confirming correct ET tube placement, but is also less reliable than waveform capnography. The presence of vapor mist in the ET tube during exhalation should not be relied upon solely; it can occur with both tracheal and esophageal intubation.

An 16-year-old male presents with acute respiratory distress. His mother tells you that he recently lost his job. He is conscious and alert, but obviously anxious. He has a respiratory rate of 40 breaths/min and an oxygen saturation of 98% on room air. Further assessment reveals carpopedal spasms to his hands. Initial management for this patient should include: A) applying high-flow oxygen via nonrebreathing mask. B) providing coaching to slow the patient's breathing. C) administering a sedative drug to help calm the patient. D) applying an oxygen mask with the flow rate set at 2 L/min.

B On the basis of the patient history and physical findings, this case is consistent with an anxiety attack and hyperventilation syndrome. These patients initially need emotional support and respiratory coaching. If the patient's respirations do not slow down after a reasonable period of coaching, you should consider the presence of another underlying cause, such as hypoxemia, and administer high-flow oxygen. Rebreathing into a paper bag or any other action that causes the patient to rebreathe his or her own carbon dioxide (ie, face mask with low-flow oxygen) is potentially dangerous and should be avoided. The causes of hyperventilation are many; acute anxiety is but one of them. If you are able to successfully control the patient's breathing, yet he is still experiencing anxiety, it would not be unreasonable to administer a sedative drug, such as Valium, provided that his vital signs are stable and his breathing remains adequate.

After determining that your unresponsive patient has a severe (complete) upper airway obstruction caused by a foreign body, you should: A) reposition the airway. B) perform chest compressions. C) reattempt to ventilate the patient. D) administer subdiaphragmatic thrusts.

B Once you have determined that your unresponsive patient has a severe (complete) foreign body airway obstruction (eg, you have attempted to ventilate, repositioned the airway, and still cannot ventilate), you should perform 30 chest compressions, open the airway and visualize the mouth, and remove the foreign body only if you can see it. Subdiaphragmatic thrusts (Heimlich maneuver) are indicated for responsive adults and children with a severe foreign body airway obstruction.

After intubating a cardiac arrest patient, you observe a capnography reading that is steadily decreasing. During each ventilation, you see obvious bilateral chest rise and ventilatory compliance is good. You should: A) give 1 mEq/kg of sodium bicarbonate. B) reevaluate the effectiveness of your CPR. C) hyperventilate the patient at 24 breaths/min. D) perform tracheobronchial suctioning at once.

B Quantitative waveform capnography should be used to assess initial advanced airway placement, as well as to monitor ongoing placement. In addition, capnography can be used as a measure of perfusion. If you note a steady decline in the PETCO2, especially if it falls below 10 mm Hg, your initial action should be to reevaluate the effectiveness of your CPR and attempt to improve the quality of chest compressions. In the context of cardiac arrest, a falling PETCO2 indicates decreased carbon dioxide return to the lungs, which may be the result of inadequate CPR. Hyperventilation should be avoided as this has been shown to impair venous return (and cardiac output) secondary to increased intrathoracic pressure. There is no indication in this scenario that the ET tube needs to be suctioned; ventilations are producing obvious chest rise and ventilation compliance is good. Routine administration of sodium bicarbonate during cardiac arrest should be avoided; ensure adequate ventilation and chest compressions first!

A patient with a reactive lower airway disease would be expected to present with: A) inspiratory stridor. B) expiratory wheezing. C) severe hypocarbia. D) pleuritic chest pain.

B Reactive lower airway disease includes any condition associated with bronchospasm (ie, asthma, bronchiolitis). A hallmark sign of bronchospasm is wheezing - a high-pitched whistling sound that indicates air movement through narrowed bronchioles. Stridor is a high-pitched sound heard during inhalation and indicates obstruction of the upper airway due to swelling or a foreign body. Because bronchospasm impairs pulmonary respiration, the patient would become hypercarbic, not hypocarbic. Pleuritic (sharp) chest pain is common in patients with a spontaneous pneumothorax or pulmonary embolism; it is not a common finding in patients with reactive airway disease.

Which of the following is MOST consistent with inadequate breathing in an adult? A) Expiratory wheezing and pink, moist skin B) 14 breaths/min with reduced tidal volume C) Eupneic respirations and warm, dry skin D) Audible rhonchi and flushed, warm skin

B Respirations of 14 breaths/min fall within the normal range for an adult. However, if the patient's breathing is shallow (reduced tidal volume), pulmonary respiration will become inadequate and minute volume will decrease. When evaluating ventilation adequacy, it is important to note the rate, depth, and regularity of breathing; do not rely solely on one parameter. The term eupnea is defined as a normal rate, depth, and regularity of breathing. Other signs of inadequate breathing include an irregular pattern of inhalation and exhalation; cool, pale, clammy skin; cyanosis (a later sign); and a decreased level of consciousness.

Snoring respirations are MOST rapidly corrected by: A) inserting a simple airway adjunct. B) manually maneuvering the head. C) suctioning of the oropharynx. D) placing the patient on his or her side.

B Snoring (sonorous) respirations indicate partial airway obstruction by the tongue. The quickest way to correct this is to perform a head tilt-chin lift maneuver, or jaw-thrust maneuver if trauma is suspected. If secretions are in the airway, suction the oropharynx after manually opening the airway. A simple airway adjunct (oral or nasal airway) should be inserted to assist in maintaining airway patency after manually opening the airway and suctioning the oropharynx if needed. Placing a patient in a recumbent (on their side) position is appropriate if he or she has a decreased level of consciousness, is uninjured, and is breathing adequately.

When attempting visualization of the vocal cords with a curved laryngoscope blade, you should: A) place the tip of the blade directly under the epiglottis and then lift with the long axis of the handle. B) place the tip of the blade in the vallecula, and lift the jaw, tongue, and blade gently at a 45° angle. C) insert the blade into the left side of the mouth, displace the tongue to the right, and then gently lift. D) insert the blade in the midline of the mouth and then gently lift when the tip of the blade is in the vallecula.

B The curved blade is designed to fit into the vallecular space, indirectly lift the epiglottis, and expose the vocal cords. The straight blade is placed under the epiglottis, directly lifting it to expose the vocal cords. Regardless of which blade you use, the proper technique of laryngoscopy involves inserting the blade in the right side of the mouth, displacing the tongue to the left, and gently lifting - not prying - at a 45° angle.

An older woman presents with respiratory distress. She is conscious and alert and is able to answer your questions with slight difficulty. Her respirations are 24 breaths/min and labored and her oxygen saturation is 89%. Further assessment reveals slight cyanosis around her mouth. You should: A) apply a nasal cannula at 4 to 6 L/min. B) apply a nonrebreathing mask at 15 L/min. C) assist her breathing with a bag-mask device. D) sedate the patient and perform intubation.

B The fact that the patient is conscious and alert and able to answer your questions, albeit with slight difficulty, indicates that her airway is patent and her breathing is adequate. However, cyanosis and a low oxygen saturation are clinical indicators of hypoxemia and should be treated with high-flow oxygen. In this patient, it would be appropriate to apply a nonrebreathing mask at 15 L/min and closely monitor the adequacy of her breathing. If her level of conscious deteriorates and she shows other signs of inadequate breathing (ie, fast or slow breathing rate, shallow depth of breathing [reduced tidal volume]), you should assist her ventilations with a bag-mask device. This patient is not a candidate for rapid-sequence intubation because she is able to maintain her own airway.

You are attempting to ventilate a patient with a bag-mask device and do not see the chest visibly rise. What should your initial course of action be? A) Switch to a pocket face mask. B) Reposition the patient's head. C) Insert a supraglottic airway device. D) Suction the oropharynx for 15 seconds.

B The first step that should be taken if the chest does not visibly rise with artificial ventilation is to reposition the head to ensure that the tongue is not blocking the airway. If there are secretions in the mouth, suction the oropharynx for no more than 15 seconds (10 seconds in children; 5 seconds in infants). If there are no secretions in the airway and repositioning the head does not allow for adequate artificial ventilation, consider that the patient has a foreign body airway obstruction. Advanced airway devices (ie, ET tube, multilumen airway, supraglottic airway) are indicated for patients who are unresponsive, cannot maintain their own airway, and require prolonged ventilatory support.

A COPD patient presents with severe respiratory distress; cyanosis to the face, neck, and chest; a decreased level of consciousness; and a heart rate of 150 beats/min. The MOST appropriate treatment for this patient includes: A) high-flow oxygen via nonrebreathing mask. B) assisting ventilations with a bag-mask device. C) hyperventilation followed by orotracheal intubation. D) oxygen via nasal cannula and 6 mg of adenosine IV.

B The patient in this scenario is experiencing respiratory failure and severe hypoxia and requires immediate positive-pressure ventilatory assistance. Initially, this should be provided with a bag-mask device attached to 100% oxygen. Devices such as a nonrebreathing mask and nasal cannula deliver oxygen passively, not via positive-pressure. Patients with inadequate ventilation benefit very little from passive oxygenation. The patient may require intubation, but not before adequate preoxygenation with basic means. Do NOT hyperventilate the patient as this causes an increase in intrathoracic pressure. Because of chronic air trapping in the lungs, COPD patients are especially susceptible to pneumothoraces caused by barotrauma. Furthermore, hyperventilation impairs venous return, which can compromise cardiac output. Adenosine is not indicated; the patient's tachycardia is likely a compensatory response to severe hypoxia, not a primary cardiac dysrhythmia.

What is the approximate minute alveolar volume of a patient who breathes in 550 mL of air at a rate of 14 times per minute? A) 4.8 L B) 5.4 L C) 6.2 L D) 7.7 L

B Tidal volume is the amount of air that moves into or out of the respiratory tract per breath. Alveolar volume is the amount of air that actually reaches the lungs per breath. Minute alveolar volume, also called minute alveolar ventilation, is calculated by multiplying the patient's tidal volume, minus dead space volume, and the respiratory rate. Approximately 30% of a person's tidal volume (or about 1 mL per pound of body weight) lingers in the anatomic dead space (eg, trachea, larger bronchi) and does not reach the lungs to participate in gas exchange. Therefore, a patient with an alveolar volume of 385 mL (550 - 165 [30% of 550] = 385) and a respiratory rate of 14 breaths/min has an approximate minute alveolar volume of 5.4 L (385 × 14 = 5,390 mL [5.4 L]).

A 21-year-old man is unresponsive and has shallow, gurgling respirations. After manually opening his airway, the paramedic should: A) begin assisting ventilations. B) suction the oropharynx. C) prepare for intubation. D) attach a pulse oximeter.

B To effectively manage a patient's airway, you must ensure that it is open and clear of foreign bodies, blood, or other secretions. The presence of gurgling is an indication that the airway contains secretions; therefore, the patient's oropharynx must be suctioned prior to any further interventions. Aspiration increases mortality significantly!

When performing tracheobronchial suctioning on an adult, it is important to: A) coat the tip of the catheter with a petroleum-based lubricant. B) monitor the patient's oxygen saturation and cardiac rhythm. C) premedicate the patient with atropine to prevent bradycardia. D) hyperventilate the patient before inserting the suction catheter.

B Tracheobronchial (endotracheal) suctioning not only removes secretions from the ET tube, but also can remove oxygen from the body. Therefore, it is important to monitor the patient's oxygen saturation and cardiac rhythm during the procedure. A sudden increase in heart rate and/or decrease in oxygen saturation indicates hypoxemia and the need to abort the suction attempt and resume ventilations. If your patient is adequately preoxygenated before the suction attempt, the incidence of hypoxemia can be greatly reduced. Do not hyperventilate the patient, however, as this may impair venous return to the heart and cause hypotension. The tip of the flexible suction catheter should be coated with a water-soluble lubricant; a petroleum-based substance would likely obstruct the ET tube. Atropine is not indicated prior to tracheobronchial suctioning; hypoxemia in the adult would likely manifest with tachycardia, not bradycardia.

An unresponsive trauma patient has sonorous respirations and blood draining from the corner of his mouth. What should be your FIRST action? A) Suction the oropharynx. B) Perform a jaw-thrust maneuver. C) Ventilate with a bag-mask device. D) Insert an oropharyngeal airway.

B When caring for any unresponsive patient, you must first open their airway. In the trauma patient, this involves using the jaw-thrust maneuver. The head tilt-chin lift maneuver should be used if the jaw-thrust does not adequately open the airway. After opening the patient's airway, ensure that it is clear of any foreign bodies or secretions and suction the oropharynx as necessary. After the airway has been manually opened and cleared of any foreign bodies or secretions, a simple airway adjunct should be inserted to help maintain airway patency. After securing a patent airway, administer oxygen or assist ventilations, as dictated by the patient's ventilation and oxygenation status.

You are ventilating an apneic, intubated patient and note that his ETCO2 reading, per waveform capnography, is 56 mm Hg. You should: A) attach a colorimetric ETCO2 detector. B) increase the rate of ventilations. C) extubate the patient immediately. D) decrease the rate of ventilations.

B When ventilating an apneic patient with spontaneous circulation (eg, a pulse), your goal is to maintain an end-tidal CO2 (ETCO2) reading of 35 to 45 mm Hg. An increasing ETCO2 indicates excess CO2 in the patient's exhaled air, and should be treated by increasing the rate of your ventilations. A low ETCO2 reading - again, in an apneic patient with spontaneous circulation - indicates a low amount of CO2 in exhaled air; therefore, you should slow your rate of ventilations accordingly. The sudden loss of a capnographic waveform and LED reading indicates that the ET tube has become dislodged and is no longer in the trachea. If inadvertent extubation occurs, remove the ET tube immediately and resume bag-mask ventilations. The colorimetric ETCO2 detector is a qualitative device that simply indicates the presence of carbon dioxide during exhalation; it is less reliable than quantitative waveform capnography.

Which of the following clinical presentations is MOST indicative of a patent airway? A) Perioral cyanosis; weak stridor B) Flushed, moist skin; weak cough C) Diaphoresis; tachycardia; gagging D) Pallor; confusion; audible wheezing

C A patent (open) airway is one that does not impede airflow into and out of the lungs. Gagging is a forceful muscular contraction of the pharyngeal muscles and the glottis. This reaction is automatic when something touches an area deep in the oral cavity - that is, when the gag reflex is stimulated. The presence of gagging indicates an intact gag reflex. Coughing is a forceful exhalation produced with a greater than normal volume of air. A person with a weak cough, which could indicate an airway obstruction or suppression of the cough reflex by drugs or trauma, is at serious risk of aspiration. Weak stridor is a sign of a marked reduction in airflow due to severe swelling of the upper airway. Cyanosis and weak stridor clearly indicate a jeopardized airway. Audible wheezing - that is, wheezing that can be heard without a stethoscope - indicates airflow obstruction in the lower airway. In the presence of an altered mental status, audible wheezing indicates significant hypoxemia.

A firefighter was exposed to smoke during a structure fire. He is conscious, alert, and oriented, but is experiencing respiratory distress. His oxygen saturation is 91% on room air and his heart rate is rapid and strong. Which of the following is the MOST appropriate initial means of oxygenating this patient? A) Nasal cannula set at 3 L/min B) Simple face mask set at 10 L/min C) Nonrebreathing mask set at 12 to 15 L/min D) Bag-mask device with high-flow oxygen

C A room air oxygen saturation reading of 91% indicates mild hypoxemia. Because the patient is conscious, alert, and oriented and is not showing outward signs of inadequate breathing, the most appropriate initial means of providing oxygenation involves applying a nonrebreathing mask with the flow rate set at 12 to 15 L/min. You must closely monitor the patient's breathing adequacy and be prepared to assist his ventilations with a bag-mask device.

Which of the following clinical presentations is MOST indicative of a severe upper airway obstruction? A) Anxiety and a forceful cough B) Flushed skin and tachycardia C) Weak cough and cyanosis D) Ability to speak in broken sentences

C A severe airway obstruction is characterized by an inability to speak, minimal or absent air movement, extreme anxiety, decreased level of consciousness (ie, confusion, lethargy, unresponsive), falling oxygen saturation, and cyanosis. If the patient is coughing, it is weak and ineffective.

Which of the following statements regarding Cheyne-Stokes respirations is correct? A) Cheyne-Stokes respirations always indicate increased intracranial pressure and brainstem dysfunction. B) It is common to observe Cheyne-Stokes respirations in patients who have overdosed on narcotic drugs. C) They are not considered ominous unless grossly exaggerated or in the context of a traumatic brain injury. D) They are characterized by an irregular pattern, rate, and depth of respiration with intermittent apneic periods.

C Cheyne-Stokes respirations are more of a high-brain function. Many deep sleepers or intoxicated people exhibit this respiratory pattern. The depth of breathing (or volume of snoring) gradually increases, then decreases (crescendo-decrescendo), followed by an apneic period. Despite their abnormal appearance, Cheyne-Stokes respirations are not considered ominous unless they are grossly exaggerated or occur in the context of a traumatic brain injury. Biot's respirations, also called ataxic respirations, are characterized by an irregular pattern, rate, and depth of breathing with intermittent apneic periods; they indicate a severe brain injury. Overdose of a narcotic (opiate) drug, such as heroin or morphine, would be expected to cause slow, shallow breathing secondary to depression of the respiratory centers in the brain.

A 60-year-old female presents with acute respiratory distress. The patient has a tracheostomy tube in place, but is able to breathe spontaneously. She is conscious, but restless. Her heart rate is 120 beats/min and her oxygen saturation is 82%. You should: A) carefully remove the tracheostomy tube and replace it with a new one. B) attach a bag device to the tracheostomy tube and mildly hyperventilate her. C) assess for secretions in the tracheostomy tube and suction the tube if needed. D) place an oxygen mask over the tracheostomy tube and reassess the patient.

C Acute respiratory distress in the patient with a tracheostomy tube is often the result of thick secretions or a mucous plug in the tube. Therefore, you should immediately assess the tracheostomy tube and determine if suctioning is required. Never routinely remove a tracheostomy tube; there is no guarantee that you will be able to replace it! If the patient's clinical condition does not improve after suctioning the tracheostomy tube - especially if signs of inadequate ventilation are noted (eg, reduced tidal volume, rapid or slow rate) - you should assist the patient's ventilations; however, you should not hyperventilate. Hyperventilation impedes venous return to the heart (preload) secondary to increased intrathoracic pressure. Placing an oxygen mask over the tracheostomy tube is appropriate only after you have ensured that the tube is not obstructed and the patient is breathing adequately.

A 60-year-old woman presents with difficulty breathing. She is conscious and alert, but anxious, and tells you that she was suddenly awakened with the feeling that she was suffocating. She has dried blood on her lips and cyanosis around her mouth. Her heart rate is 120 beats/min and her oxygen saturation is 89%. Your initial action should be to: A) attach a cardiac monitor. B) auscultate breath sounds. C) apply supplemental oxygen. D) administer a bronchodilator.

C Although the patient is conscious and able to converse, she is clearly showing signs of hypoxemia (anxiety, oxygen saturation of 89%, tachycardia). Therefore, she should be given supplemental oxygen as soon as possible. You must be prepared to assist her ventilations if her level of consciousness deteriorates or her breathing becomes inadequate. If there is evidence of pulmonary edema (ie, rales, rhonchi), you should consider applying continuous positive airway pressure (CPAP).

The normal partial pressure of oxygen in arterial blood is: A) 35 to 45 mm Hg. B) 60 to 80 mm Hg. C) 80 to 100 mm Hg. D) 100 to 120 mm Hg.

C Arterial blood should have a high partial pressure of oxygen because it has been reoxygenated in the lungs. The normal partial pressure of oxygen in arterial blood (PaO2) is 80 to 100 mm Hg. An arterial PaO2 of less than 80 mm Hg indicates tissue hypoxia. The normal partial pressure of carbon dioxide in arterial blood (PaCO2) is 35 to 45 mm Hg.

A 30-year-old man overdosed on codeine and has respirations of 6 breaths/min and shallow. Which of the following conditions will he develop initially? A) Metabolic alkalosis B) Metabolic acidosis C) Respiratory acidosis D) Respiratory alkalosis

C As respirations decrease in rate and depth (tidal volume), carbon dioxide is retained by the body. This leads to an initial state of respiratory acidosis. If left untreated, metabolic acidosis will result as the cells of the body begin producing lactic acid secondary to anaerobic metabolism. Remember that the initial treatment for acidosis, regardless of the underlying cause, is adequate ventilation.

Initial management of an unresponsive 20-year-old patient with respirations of 14 breaths/min and adequate depth should include: A) performing immediate intubation to protect the patient's airway. B) inserting an airway adjunct and providing assisted ventilations. C) inserting an airway adjunct and giving oxygen via nonrebreathing mask. D) inserting an airway adjunct and suctioning the mouth every 30 seconds.

C Because the patient is unresponsive, an airway adjunct (oral or nasal airway) should be inserted, which, along with manual airway positioning, will help maintain airway patency. The patient in this scenario has adequate ventilation (14 breaths/min with adequate depth [tidal volume]); therefore, high-flow oxygen via nonrebreathing mask would be the most appropriate initial approach. It is essential to continue to monitor the patient for signs of inadequate ventilation, and be prepared to assist ventilations if needed. Advanced airway management should be considered if the unresponsive patient requires ventilatory support, especially if the need for ventilatory support will be prolonged.

You are ventilating an intubated patient who has been in cardiac arrest for approximately 15 minutes. Despite the presence of bilaterally equal breath sounds, quantitative capnography persistently reads less than 10 mm Hg. This MOST likely indicates that: A) you are ventilating the patient too fast. B) you need to increase the ventilatory rate. C) the patient is not producing carbon dioxide. D) the body's cells are rapidly metabolizing oxygen.

C Before carbon dioxide can be eliminated from the body and detected with capnography, the body must be able to metabolize oxygen in order to produce carbon dioxide (aerobic metabolism). Patients in cardiac arrest - especially prolonged cardiac arrest - are severely acidotic; the cells are metabolizing carbon dioxide and producing lactic acid (anaerobic metabolism). As a result, minimal (or no) carbon dioxide is being produced and returned to the lungs. This would result in a persistently low (< 10 mm Hg) end-tidal CO2 (ETCO2) reading. However, when ventilating an intubated patient who has a pulse - that is, he or she is perfusing and is producing carbon dioxide - a low (< 35 mm Hg) ETCO2 reading indicates that you are ventilating too fast and are eliminating too much CO2 from the body. Therefore, you should decrease the rate of ventilation. By contrast, if the intubated patient with a pulse displays an ETCO2 reading that is too high (> 45 mm Hg), he or she has excess carbon dioxide that needs to be eliminated; therefore, you should increase the rate of ventilation.

Which of the following airway devices is MOST appropriate to use in a deeply unresponsive intoxicated patient? A) Perilaryngeal airway B) Nasopharyngeal airway C) Endotracheal tube D) Laryngeal mask airway

C Deeply unresponsive patients, especially those who are intoxicated, are at high risk of regurgitation because their stomach is full of alcohol. The endotracheal tube virtually eliminates the risk of aspiration because it isolates the trachea. The laryngeal mask airway (LMA) is a useful alternative airway device that has been shown to provide adequate ventilation; however, it does not eliminate the risk of aspiration. Perilaryngeal airways, such as the CobraPLA, are also useful alternative airway devices and provide adequate ventilation; however, like the LMA, they do not eliminate the risk of aspiration. Obviously, a nasopharyngeal airway will not protect the airway from aspiration; like the oropharyngeal airway, it keeps the tongue away from the posterior pharynx.

When ventilating an intubated patient in cardiac arrest, which of the following end-tidal CO2 (PETCO2) findings indicates return of spontaneous circulation? A) Progressive decrease in PETCO2 B) Complete absence of a PETCO2 reading C) Abrupt and sustained increase in PETCO2 D) Abrupt and sustained decrease in PETCO2

C During cardiac arrest and other low perfusion states, decreased amounts of carbon dioxide are returned to the lungs due to anaerobic metabolism and lactic acidosis. This explains why you typically see progressively decreasing end-tidal CO2 (PETCO2) readings in cardiac arrest patients. However, if return of spontaneous circulation (ROSC) occurs, increased amounts of carbon dioxide are returned to the lungs, resulting in an abrupt and sustained increased in PETCO2 that is typically equal to or greater than 40 mm Hg. A complete absence of a PETCO2 reading and capnographic waveform indicates that the endotracheal tube is not in the trachea.

Which of the following processes occurs during inhalation? A) The diaphragm contracts and ascends, intrathoracic pressure decreases, and air enters the lungs via positive pressure. B) The intercostal muscles and diaphragm contract, intrathoracic pressure increases, and air passively enters the lungs. C) The diaphragm contracts and descends, intrathoracic pressure decreases, and air enters the lungs via negative pressure. D) The intercostal muscles relax, the ribcage expands, intrathoracic pressure increases, and air enters the lungs via positive pressure.

C During inhalation - an active process - the diaphragm contracts and descends and the intercostal muscles contract. These processes cause an increase in the vertical and horizontal dimensions of the thoracic cavity. As a result, intrathoracic pressure falls and air is drawn into the lungs via negative pressure. Exhalation is a passive process that occurs when the diaphragm and intercostal muscles relax and air exits the lungs passively. Positive pressure ventilation is the act of forcing air into the lungs and is not a part of normal breathing; it is provided by the paramedic in the form of artificial ventilation (eg, rescue breathing).

While intubating a 44-year-old man in respiratory arrest, you note that his pulse rate increases during the procedure. What should you do? A) Discontinue the intubation attempt and hyperventilate the patient for 2 to 3 minutes. B) Continue your intubation attempt as your partner performs a carotid sinus massage. C) Recognize this as a normal response during intubation and monitor the pulse rate. D) Complete the intubation attempt and administer adenosine after the tube is secure.

C Endotracheal intubation typically causes stimulation of the sympathetic nervous system; therefore, it is common to see increases in both the pulse rate and blood pressure during the procedure. This is usually handled well in most patients, provided that they do not have concomitant head injury with increased intracranial pressure (ICP). In cases such as this, lidocaine can be considered, which has been shown, albeit through an unknown mechanism, to transiently blunt a sudden rise in ICP during intubation. Routine hyperventilation should be avoided as it has been shown to increase intrathoracic pressure and impair venous return to the heart. Ventilate the apneic adult at a rate of 10 to 12 breaths/min (one breath every 5 to 6 seconds).

Which of the following is associated with insufficient carbon dioxide elimination? A) Acute hyperventilation B) Tachypnea and hyperpnea C) Bradypnea and hypopnea D) A venous PCO2 of 46 torr

C Hypopnea (shallow breathing [reduced tidal volume]) and bradypnea (slow respirations) result in both insufficient oxygen intake and insufficient carbon dioxide (CO2) elimination and should be treated with positive-presure ventilation to increase oxygen intake and facilitate CO2 elimination. A patient with tachypnea (rapid breathing) and hyperpnea (deep breathing) eliminates adequate - often increased - amounts of CO2 from the body. Hyperventilation occurs when CO2 elimination exceeds CO2 production. A venous PCO2 of 46 torr represents a normal level, and thus, adequate CO2 elimination.

You are transporting a patient with respiratory distress when he suddenly becomes unresponsive and does not appear to be breathing. The paramedic should: A) insert a supraglottic airway device. B) open the airway and give 2 rescue breaths. C) assess for a carotid pulse for up to 10 seconds. D) prepare for immediate endotracheal intubation.

C If an adult patient becomes unresponsive and apneic (as evidenced by an absence of obvious chest movement) in your presence or is found to be unresponsive and apneic, you should assess for a carotid pulse for 5 to 10 seconds. If the patient has a pulse, open the airway and provide rescue breathing at a rate of 10 to 12 breaths/min (one breath every 5 to 6 seconds). If the patient does not have a pulse, immediately perform 30 chest compressions, then open the airway and provide two rescue breaths. The technique of look, listen, and feel is no longer recommended because it delays the initiation of chest compressions in patients who are in cardiac arrest. Unless airway patency and adequate ventilation cannot be maintaned with less invasive means (ie, oral/nasal airway and bag-mask ventilation), advanced airway management should not be an immediate priority during the initial management of a patient in respiratory or cardiac arrest.

After placing an endotracheal tube in a cardiac arrest patient, large amounts of vomitus immediately begin flowing out of the tube. You should: A) withdraw the ET tube just until the flow of vomitus stops and inflate the distal cuff with 5 to 10 mL of air. B) immediately remove the ET tube, resume bag-mask ventilations, and reattempt intubation after 2 minutes. C) leave the ET tube in place, fold it to the side so the vomitus can drain, and resume bag-mask ventilations. D) apply posterior pressure to the cricoid cartilage to compress the esophagus and stop the flow of vomitus.

C If inadvertant esophageal intubation occurs and vomitus begins flowing out of the ET tube, you should leave the ET tube in place, inflate the distal cuff with more air than normal (perhaps as much as 20 to 30 mL), fold the ET tube to the side to allow the vomitus to drain, and resume bag mask ventilations. If you remove the ET tube when the patient is regurgitating, then you have virtually assured aspiration. With proper technique, you can maintain an adequate mask seal over the folded ET tube in order to ventilate the patient with the bag-mask device. Applying posterior cricoid pressure may compress the esophagus, but will NOT compress the ET tube that is in the esophagus. Consider attaching suction to the end of the ET tube to decompress the stomach.

You have attempted orotracheal intubation on a cardiac arrest patient, but were unsuccessful after two attempts. When you resume bag-mask ventilations, you are unable to maintain an adequate mask-to-face seal. What should you do? A) Perform a surgical cricothyrotomy. B) Perform nasotracheal intubation instead. C) Insert a multilumen or supraglottic airway device. D) Have your partner attempt to intubate the patient.

C If you are unable to successfully perform orotracheal intubation, and you cannot provide effective ventilations with the bag-mask device, the quickest and most practical approach would be to insert an alternative airway device, such as a multilumen airway (eg, Combitube) or a supraglottic airway (eg, LMA, King LT, CobraPLA), and resume ventilations as soon as possible. These devices are easy to insert, have been shown to provide better ventilation than the bag-mask device, and are much safer than performing a surgical (open) cricothyrotomy. Do not develop tunnel-vision and fixate on orotracheal intubation when alternative airway devices are available. Remember, the goal is to ventilate, not intubate. Nasotracheal intubation is contraindicated in this patient because he is apneic.

During an intubation attempt, you are having difficulty viewing the patient's vocal cords. Which of the following actions would MOST likely help? A) Flex the patient's neck and pull up on the chin. B) Switch to a laryngoscope handle that is larger. C) Ask your partner to manipulate the external larynx. D) Place the patient's head in a hyperextended position.

C If you experience difficulty when attempting to view a patient's vocal cords during intubation, you can ask your partner to perform the BURP maneuver (external laryngeal manipulation), which involves applying Backward, Upward, and Rightward Pressure to the larynx; this technique often improves your laryngoscopic view of the vocal cords. You can also insert a gum elastic bougie through the vocal cords, stroke the wall of the trachea with the bougie (you should feel the "bumps" of the tracheal wall), and then feed the ET tube over the bougie and into the trachea. The patient's head should be in the sniffing position during intubation, not hyperextended or flexed. Regardless of the size laryngoscope handle that you use, the technique of laryngoscopy is the same; switching to a larger (or smaller) handle will likely not help improve your laryngoscopic view of the vocal cords.

Immediately after placing an endotracheal tube in the trachea of an adult, the paramedic should: A) attach the bag-mask device and ventilate. B) attach an end-tidal carbon dioxide detector. C) inflate the distal cuff with 5 to 10 mL of air. D) secure the tube with an appropriate device.

C Immediately following placement of an endotracheal tube, the paramedic should inflate the distal cuff with 5 to 10 mL of air and detach the syringe. This is a critical step in the intubation procedure because the distal cuff protects the lungs from aspiration. After the cuff is inflated and the stylet has been removed, attach an end-tidal CO2 detector (quantitative waveform capnography should be used) in between the bag-mask device and ET tube, begin to ventilate the patient, and auscultate over the epigastrium and the apices and bases of both lungs. After confirming proper ET tube placement, secure the tube with an appropriate device; a commercially-manufactured tube-securing device is recommended.

You are ventilating a severely dehydrated apneic 70-year-old male with a history of end-stage emphysema. In order to minimize the risk of lowering his cardiac output and blood pressure, you should: A) use a manually triggered ventilation device. B) hyperventilate the patient at 20 to 24 breaths/min. C) adjust the ventilation rate to allow complete exhalation. D) use a device that provides positive-end expiratory pressure.

C In patients with severe COPD (ie, end-stage emphysema) and increased resistance to exhalation, you should attempt to prevent air trapping as this may cause inadvertent generation of intrinsic positive end-expiratory pressure (also called "auto-PEEP"). In hypovolemic patients - as with your severely dehydrated patient - auto-PEEP may significantly reduce cardiac output and blood pressure. Adjusting the ventilation rate to approximately 6 to 8 breaths/min - which will allow for complete exhalation - can prevent this. Manually-triggered ventilation devices (eg, demand valve) should not be used in any patient with pulmonary air trapping; use of such devices may result in widespread alveolar rupture and/or a pneumothorax.

You have inserted an oral airway in an apneic patient and are ventilating him with a bag-mask device when he suddenly vomits. After removing the oral airway, you should: A) suction his oropharynx. B) apply cricoid pressure. C) turn him onto his side. D) insert a Combitube.

C Keeping in mind that mortality increases significantly if aspiration occurs, you must immediately remove the oral airway and turn the patient onto his side - which will facilitate drainage of vomitus from his mouth - and then suction his oropharynx. Do not suction the unprotected airway of a patient while he or she is supine; this only increases the likelihood of aspiration. Posterior cricoid pressure (Sellick maneuver) is no longer a recommended technique as it has been shown to impair ventilation and does not prevent gastric distention and regurgitation as was once thought. A Combitube or other airway device designed to enter the esophagus should not be inserted if the patient is actively vomiting.

A 50-year-old man with a self-inflicted gunshot wound to the face is apneic. He has multiple fractures of the mandible, massive soft tissue damage, and severe oropharyngeal bleeding. Which of the following methods of airway control will be MOST effective for this patient? A) Orotracheal intubation B) Nasotracheal intubation C) Surgical cricothyrotomy D) Oral airway and bag-mask ventilation

C Managing the airway of a patient with massive maxillofacial trauma can present a challenge to even the most experienced paramedic. In this case, orotracheal intubation would be extremely difficult because of the blood in the patient's mouth and his mandibular fractures (orotracheal intubation relies on a stable mandible). Nasotracheal intubation is contraindicated because the patient is apneic. Bag-mask ventilations will likely be ineffective because maintaining a mask seal will be extremely difficult. Furthermore, bag-mask ventilation in a patient with active oropharyngeal bleeding will force blood into the lungs, causing the patient to aspirate. Although considered as a last resort, this patient is a candidate for a surgical (open) cricothyrotomy.

You would MOST likely encounter bradypnea in a patient who: A) ingested salicylates. B) is acutely hypoxemic. C) has metabolic alkalosis. D) took 5 mg of diazepam.

C Metabolic alkalosis (pH > 7.45) often results in periods of bradypnea (abnormally slow respirations). This occurs due to compensation by the respiratory buffer system, which is attempting to retain carbon dioxide and hydrogen ions in order to lower the pH. Five milligrams of diazepam (Valium) is a therapeutic dose; CNS depression (ie, bradypnea, bradycardia, hypotension) would be unlikely at this dose, unless the patient co-ingested another CNS depressant (eg, opiates). When the body's chemoreceptors sense increased arterial CO2 levels (hypercarbia) or decreased O2 levels (hypoxemia), the respiratory centers in the brain send more messages to the respiratory muscles; as a result, respirations increase (tachypnea) in order to bring in more oxygen and eliminate more carbon dioxide. A patient who ingested salicylates (ie, acetylsalicylic acid [aspirin, ASA]) would present with tachypnea because the respiratory buffer system is attempting to eliminate excess hydrogen ions by increasing the rate and depth of breathing.

When administering oxygen via nasal cannula during a long-range transport, you should: A) set the flow rate to at least 4 L/min. B) vary the liter flow from 1 to 6 L/min. C) attach an oxygen humidifier. D) ensure that the patient is supine.

C Oxygen that is delivered nasally, especially over a prolonged period of time, can cause drying and irritation of the nasal mucosa; therefore, an oxygen humidifier should be attached. The appropriate oxygen flow rate for the nasal cannula is 1 to 6 L/min. A semi-sitting (semi-Fowler) position is preferable for most patients, especially those who are experiencing breathing difficulty.

Which of the following is the MOST appropriate and effective method of oxygenating a semiconscious patient with slow, shallow breathing? A) Nasal cannula at 1 to 6 L/min B) Nonrebreathing mask at 15 L/min C) Bag-mask device and high-flow oxygen D) Oxygen-powered transport ventilator

C Patients with slow, shallow (reduced tidal volume) breathing are not taking in sufficient amounts of air to maintain adequate minute volume and require some form of positive-pressure ventilation. This is especially true for patients who are semiconscious or unconscious. Assist the patient's ventilations with a bag-mask device attached to high-flow oxygen. Neither the nasal cannula nor the nonrebreathing mask will improve the patient's tidal volume, and would therefore be of little benefit to the patient with inadequate ventilation. Transport ventilators are used for unresponsive apneic patients, generally after they have been intubated.

Assessment of a patient with acute respiratory distress reveals that he is conscious and alert, but wheezing on exhalation. In addition to oxygen, management should include: A) pharmacologically assisted intubation. B) administration of 3 to 5 mg of epinephrine subcutaneously. C) administration of an inhaled beta-2 agonist medication. D) administration of a beta-blocker, such as metoprolol tartrate.

C Selective beta-2 agonists, such as albuterol (Ventolin, Proventil) or metaproterenol sulfate (Alupent), are typically given via a nebulizer for patients with reactive airway diseases (ie, asthma, bronchiolitis, etc) to promote bronchodilation and improve ventilation. Continuous positive airway pressure (CPAP) has also shown to be effective in treating patients with acute bronchospasm, especially when used in conjunction with an in-line nebulizer and beta-2 agonist. Epinephrine 1:1,000, in a dose of 0.3 to 0.5 mg (0.3 to 0.5 mL) subcutaneously, could also be administered to patients with acute bronchospasm. A patient with bronchospasm, as evidenced by wheezing, must never be given a beta-blocker as this may exacerbate their condition and could lead to respiratory failure or arrest. Some patients with severe refractory bronchospasm may require intubation; this is especially true if their level of consciousness deteriorates. However, the patient in this scenario is conscious and alert.

Which of the following signs is unique to a severe foreign body upper airway obstruction? A) Anxiety B) Tachycardia C) Perioral cyanosis D) Difficulty speaking

C Signs of a severe (complete) upper airway obstruction include a weak, ineffective cough; an inability to speak; and cyanosis, which indicates decreased blood oxygen levels. Tachycardia and anxiety can occur with both mild (partial) and severe airway obstructions. Signs of a mild foreign body airway obstruction include a strong cough, difficulty speaking, normal skin color, and a normal level of consciousness.

Which of the following clinical signs is MOST indicative of adequate breathing? A) Reduced tidal volume B) Tachypnea and hypopnea C) Pink oral mucous membranes D) Unilateral chest wall movement

C Signs of adequate breathing include bilateral chest wall movement, adequate depth (tidal volume), pink skin (including mucous membranes), and an adequate rate. Tachypnea (rapid breathing) and hypopnea (shallow breathing [reduced tidal volume]) will result in a decreased minute volume.

You are assisting the ventilations of an adult with a bag-mask device. The patient is responsive to pain only and has a heart rate of 140 beats/min. Which of the following signs would indicate that your assisted ventilations are inadequate? A) Equal chest wall excursion B) Minimal abdominal movement C) A marked increase in heart rate D) Increased ventilation compliance

C Signs of inadequate artificial ventilation include minimal or no rise of the chest, significant air leakage from around the mask, an increase in the patient's heart rate, cyanosis that is not resolving, a falling oxygen saturation, and decreased ventilation compliance (increased resistance when ventilating). Tachycardia is a sign of hypoxia and should begin to resolve if your assisted ventilations with high-flow oxygen are adequate.

Factors that increase the amount of energy needed for ventilation include all of the following, EXCEPT: A) surfactant deficiency. B) increased airway resistance. C) stimulation of beta-2 receptors. D) decreased pulmonary compliance.

C The amount of energy needed for normal (unassisted) ventilation in an otherwise healthy adult is only about 3% of the total body expenditure. Stimulation of beta-2 receptors would facilitate ventilation by dilating the bronchioles, and would not increase the amount of energy required for ventilation. Anything that impairs ventilation - loss of pulmonary surfactant (eg, emphysema), increased airway resistance (eg, bronchospasm), and decreased pulmonary compliance (eg, pulmonary edema, COPD) - can cause a significant increase in the amount of energy needed for ventilation, perhaps as high as 35%.

While attempting to ventilate an apneic patient with a stoma, you note minimal rise of the chest and can hear air escaping through the upper airway. You should: A) perform an emergency tracheotomy just inferior to the site of the stoma. B) hyperextend the patient's head, insert an oral airway, and reattempt to ventilate. C) pinch the nostrils closed, ensure the mouth is closed, and reattempt to ventilate. D) insert a nasopharyngeal airway and suction the stoma for no more than 5 seconds.

C The patient has likely had a partial laryngectomy and is a "partial neck breather." Because these patients are able to inhale and exhale some air via the nose and mouth, you should pinch the nostrils closed, ensure that the mouth is closed, and reattempt to ventilate. If you are still unable to achieve visible chest rise, suctioning of the stoma may be required. When ventilating a stoma patient, the head should be kept in a neutral position with the shoulders slightly elevated. This position allows for more effective ventilation.

While transporting a man with severe respiratory distress, he pulls the oxygen mask from his face and frantically attempts to get off of the stretcher. You should: A) place the patient supine and assist his ventilations. B) administer 5 mg of diazepam to calm the patient. C) apply a nasal cannula and try to calm the patient. D) hold the mask to his face and provide reassurance.

C The patient is obviously experiencing significant respiratory distress and is hypoxic, as evidenced by his extreme restlessness. In cases such as this, your most appropriate action is to attempt to calm the patient and offer oxygen with a less oppressive device, such as a nasal cannula, and closely monitor his breathing and level of consciousness. Holding the mask to the patient's face will arguably make him more anxious and increase his body's demand for oxygen. If his level of consciousness deteriorates, you should begin assisting his ventilations. CNS depressants, such as diazepam (Valium), should not be given to hypoxic patients with respiratory distress as it may cause respiratory arrest.

In contrast to the pneuomotaxic center of the medulla, the apneustic center: A) is responsible for setting a person's resting respiratory rate. B) is responsible for terminating inspiration to prevent overexpansion of the lungs. C) influences the respiratory rate by increasing the number of inspirations per minute. D) decreases its influence in times of increased demand by increasing the respiratory rate.

C The respiratory center in the medulla is divided into three regions: the respiratory rhythmicity center, the apneustic center, and the pneumotaxic center. The respiratory rhythmicity center sets the resting respiratory rate. The apneustic center influences the respiratory rate by increasing the number of inspirations per minute. Its activity is countered by the pneumotaxic center, which inhibits inspiration. In times of increased demand, the pneumotaxic center decreases its influence, thereby increasing the respiratory rate. As the chest wall expands, mechanical (stretch) receptors in the lungs send a signal to the apneustic center via the vagus nerve to inhibit the inspiratory center, and expiration occurs. This feedback loop, which combines neural and mechanical control, is called the Hering-Breuer reflex. It is a protective mechanism that terminates inspiration, thus preventing overexpansion of the lungs.

A 40-year-old woman who was recently discharged from the hospital reports a sudden onset of difficulty breathing and sharp chest pain that increases with breathing. Her skin remains cyanotic and her oxygen saturation remains low, despite high-flow oxygen. Which of the following is this patient MOST likely experiencing? A) Spontaneous pneumothorax B) Acute bacterial pneumonia C) Acute pulmonary embolism D) Acute pulmonary artery rupture

C This is a rather classic presentation of an acute pulmonary embolism, which is characterized by a sudden onset of difficulty breathing and sharp (pleuritic) chest pain. Persistent cyanosis and an oxygen saturation level that remains low despite high-flow oxygen are also common findings, especially in patients with a large pulmonary embolus. Patients who have been recently hospitalized or otherwise immobile for a prolonged period of time are at risk of developing a pulmonary embolus. This is because blood stagnates in the lower extremities, resulting in the eventual formation of a thrombus (clot), which breaks free, travels to the lungs, and lodges in a pulmonary artery.

A 60-year-old man was injured when his tractor rolled over on him. The tractor has been stabilized by rescue personnel. When you assess the man, you note that he is responsive to pain only. You should: A) apply oxygen with a nonrebreathing mask. B) assess his breathing for 5 to 10 seconds. C) open his airway with the jaw-thrust maneuver. D) begin some form of positive-pressure ventilation.

C This patient's level of consciousness is markedly diminished; you must ensure that his airway is open. The trauma patient's airway is opened by manually stabilizing his or her head and using the jaw-thrust maneuver. However, if the jaw-thrust maneuver does not adequately open the patient's airway, you should carefully perform the head tilt-chin lift manuever. After his airway is open, ensure that it is clear of secretion or foreign bodies; use suction as needed. After establishing a patent airway, assess the adequacy of his breathing and treat accordingly.

You have intubated a 33-year-old woman in cardiac arrest secondary to trauma. While auscultating her lungs, you note that breath sounds are absent over the right hemithorax. This clinical finding is MOST suggestive of: A) left mainstem bronchus intubation. B) intubation of the hypopharyngeal area. C) blood or air in the right hemithorax. D) inadvertent intubation of the esophagus.

C When breath sounds are unilaterally diminished or absent, especially following trauma, intrathoracic injury (ie, tension pneumothorax, hemothorax, hemopneumothorax) should be suspected. Anatomically, it is nearly impossible to insert the endotracheal tube too far into the left mainstem bronchus because it takes a more acute angle than the right mainstem bronchus. Inadvertent intubation of the esophagus or hypopharyngeal area would result in absent breath sounds bilaterally.

After inserting an advanced airway device in an adult patient who is in cardiac arrest, you should: A) deliver each breath over a period of 2 seconds at a rate of 20 breaths/min. B) instruct the compressor to stop compressions while you deliver ventilations. C) deliver each breath over a period of 1 second at a rate of 8 to 10 breaths/min. D) instruct the compressor to continue compressions as you hyperventilate the patient.

C When ventilating a cardiac arrest patient after an advanced airway device has been inserted (eg, ET tube, multilumen airway, supraglottic airway), do not synchronize compressions with ventilations. Perform continuous chest compressions at a rate of at least 100/min and deliver ventilations at a rate of 8 to 10 breaths/min (one breath every 6 to 8 seconds). Deliver each ventilation over a period of 1 second while observing for visible chest rise. Hyperventilating the patient should be avoided because it increases intrathoracic pressure, which can impair venous return to the heart and reduce chest compression effectiveness.

When assessing a responsive patient with a suspected pulmonary embolism (PE), it is important to remember that: A) a PE is caused by a problem in breathing mechanics. B) hypocarbia quickly develops due to hypoventilation. C) most conscious patients with a PE have cape cyanosis. D) ventilation continues, but oxygenation is inadequate.

D A pulmonary embolism (PE) occurs when a thrombus breaks free from another part of the body (often a deep vein thrombosis [DVT]) and lodges in a pulmonary artery. As a result, pulmonary gas exchange, tissue oxygenation, and perfusion are impaired. However, the patient continues to ventilate - that is, he or she continues to move air into and out of the lungs. This lack of oxygenation and perfusion, despite ongoing ventilation, creates a ventilation-perfusion mismatch. Conditions that affect the mechanics of breathing include flail chest, rib fractures, and tension pneumothorax, among others. Hypoventilation leads to hypercarbia (CO2 retention), not hypocarbia. Cape cyanosis - profound cyanosis to the upper torso, shoulders, and face - is often observed in patients with a massive PE, specifically a saddle embolus, which lodges where the main pulmonary artery branches into the left and right pulmonary arteries. Most patients with a saddle embolus are in cardiac arrest. Cape cyanosis is not commonly observed in responsive patients with a smaller PE.

Which of the following would MOST likely occur if an adult patient is breathing at a rate of 36 breaths/min and shallow? A) Tidal volume would increase. B) Minute volume would remain unchanged. C) Tidal volume and minute volume would increase. D) More air would linger in the anatomic dead space.

D A respiratory rate of 36 breaths/min and shallow (reduced tidal volume) would result in lesser amounts of air reaching the alveoli and participating in pulmonary gas exchange (pulmonary respiration). As the respiratory rate becomes faster, they also become more shallow. This would result in more air remaining in the anatomic dead space (ie, trachea, large bronchi); as a result, minute volume would decrease.

Which of the following statements regarding end-tidal CO2 (ETCO2) monitoring following endotracheal intubation is correct? A) ETCO2 monitoring is most reliable in patients with poor pulmonary perfusion. B) The ETCO2 detector is not affected by endotracheally administered medications. C) Colorimetric ETCO2 detectors provide objective data and are the preferred device. D) Quantitative waveform capnography is extremely accurate and should be used.

D According to the 2010 guidelines for CPR and emergency cardiac care (ECC), quantitative waveform capnography should be used to confirm initial ET tube placement and to monitor ongoing ET tube placement. Waveform capnography is extremely accurate and provides real-time objective data via an LED reading and a capnographic waveform. Because poor pulmonary perfusion results in decreased CO2 elimination, the ETCO2 detector is less reliable in such cases. Furthermore, elimination and detection of CO2 following endotracheally-administered drugs - most notably epinephrine - can be drastically reduced. Colorimetric ETCO2 detectors provide qualitative data; they simply detect the presence of carbon dioxide and are less reliable than waveform capnography, which provides quantitative data.

An elderly man is unresponsive and apneic. What should you do? A) Perform immediate intubation. B) Provide two rescue breaths. C) Assess his cardiac rhythm. D) Assess for a carotid pulse.

D After determining that an adult patient is unresponsive and apneic, you should assess for a carotid pulse for at least 5 seconds but no more than 10 seconds. If a pulse is present, provide rescue breathing (10 to 12 breaths/min in the adult). If a pulse is absent, begin CPR (starting with chest compressions), then open the airway and give two rescue breaths. Continue CPR and assess his cardiac rhythm as soon as possible. If you are able to effectively ventilate the patient with an oral airway and bag-mask device, intubation should not be an immediate priority. Furthermore, you should not attempt intubation until the patient has received at least 2 to 3 minutes of preoxygenation. Rare is the need to immediately intubate anyone.

Which of the following patients would benefit the MOST from continuous positive airway pressure (CPAP)? A) 39-year-old female with a cough and scattered wheezing B) 72-year-old unresponsive male with slow, irregular breathing C) 45-year-old male with anaphylaxis and severe upper airway swelling D) 61-year-old male with severe respiratory distress and diffuse crackles

D Continuous positive airway pressure (CPAP) is a noninvasive form of positive-pressure ventilation. It is used in the treatment of CHF with pulmonary edema, as evidence by respiratory distress and abnormal breath sounds (ie, rhonchi, crackles), and in patients with severe acute bronchospasm (ie, asthma). CPAP transmits positive-pressure to the lower airways, where it forces fluid from the alveoli, reexpands atelectatic alveoli, and opens the bronchioles. For acute bronchospasm, CPAP is commonly used in conjunction with an in-line nebulizer, which simultaneously delivers beta-2 agonist medications. Patients receive the maximum benefit from CPAP during exhalation because they are breathing against a positive-pressure gradient, which can be adjusted accordingly. CPAP has been shown to reduce the need for intubation and often causes rapid improvement in the patient's clinical status. CPAP is contraindicated in patients who are unable to follow verbal commands and in those who are hypoventilating (eg, slow rate, reduced tidal volume).

You are ventilating a patient with massive maxillofacial trauma after having successfully performed a needle cricothyrotomy. It is MOST important that you: A) ventilate the patient for approximately 3 to 5 minutes before attempting tracheal intubation. B) slightly extend the patient's head to ensure consistent delivery of adequate ventilations. C) monitor the patient's cardiac rhythm and oxygen saturation for signs of catheter dislodgement. D) open the release valve on the jet ventilator device just until the patient's chest visibly rises.

D Cricothyrotomy - needle or surgical - is indicated when you are unable to ventilate a patient by other, less invasive means (eg, bag-mask device, intubation). If you are able to adequately ventilate the patient after performing a needle cricothyrotomy, continue to do so and rapidly transport; you have achieved your objective. As with any other technique of ventilation, you must ensure the chest rises adequately with each ventilation. After performing a needle cricothyrotomy, this involves opening the release valve on the jet ventilator just until the chest visibly rises. Extreme care must be exercised when providing positive-pressure ventilation to any patient. Overinflation of the lungs can cause barotrauma and a resultant pneumothorax. An advantage of performing a cricothyrotomy is that it does not require manipulation of the cervical spine. This is especially beneficial in patients with massive maxillofacial trauma, in which case a cervical spine injury should be assumed. Cardiac rhythm and oxygen saturation monitoring are important when ventilating any patient - regardless of the method of ventilation. However, dislodgement of an advanced airway device would be detected quicker by monitoring for bilaterally equal breath sounds and using capnography. Do not rely solely on cardiac monitoring and pulse oximetry.

Common clinical signs of respiratory distress include all of the following, EXCEPT: A) circumoral cyanosis. B) intercostal retractions. C) pursed-lip breathing. D) a flushed appearance.

D Flushed (red) skin is not a common clinical sign of respiratory distress. It is more commonly seen in patients with a fever or in those who have been exposed to high environmental temperatures. Common signs of respiratory distress include intercostal retractions, accessory muscle use, pursed-lip breathing (especially common in patients with emphysema), and, as a later sign, cyanosis. Cyanosis around the mouth is called circumoral (or perioral) cyanosis.

An unresponsive, apneic patient has massive facial trauma, including a crushed mandible and severe oral bleeding. Orotracheal intubation has been attempted twice without success. The paramedic should proceed with: A) digital intubation. B) blind nasotracheal intubation. C) insertion of a supraglottic device. D) needle or surgical cricothyrotomy.

D In a patient with massive maxillofacial trauma and severe oral bleeding, orotracheal intubation may be extremely difficult or impossible to perform. A supraglottic airway device (ie, King LT, CobraPLA, LMA) would be of limited benefit because it does not eliminate the threat of aspiration. Digital (tactile) intubation often takes too long to perform and is frequently unsuccessful, and nasotracheal intubation is contraindicated in apneic patients. In cases where orotracheal intubation is not possible and other forms of airway management are not feasible, a needle or surgical cricothyrotomy should be performed in order to gain control over the patient's airway.

Common signs of a laryngeal fracture include all of the following, EXCEPT: A) stridor. B) dysphagia. C) hemoptysis. D) hematemesis.

D Laryngeal fractures are most often caused by blunt trauma to the anterior neck, usually following a motor-vehicle crash. You should suspect a laryngeal fracture if a trauma patient presents with painful or difficult swallowing (dysphagia), coughing up blood (hemoptysis), hoarseness, or difficulty speaking (dysphonia). As soft tissues of the upper airway swell, air movement becomes restricted; this results in stridor - a high-pitched sound heard during inhalation. Vomiting blood (hematemesis) is not a common clinical sign in patients with a laryngeal fracture, although it may be observed if the patient swallows large amounts of blood.

A responsive 20-year-old male with a suspected foreign body airway obstruction is anxious, coughing forcefully, and is able to speak with difficulty. You should: A) insert a nasopharyngeal airway to maintain airway patency. B) perform laryngoscopy and remove the obstruction with Magill forceps. C) perform subdiaphragmatic thrusts with the patient in a standing position. D) closely monitor the patient's condition and encourage him to keep coughing.

D Patients with a mild airway obstruction (eg, strong cough, adequate mental status, normal oxygen saturation, ability to speak with difficulty) are able to move enough air to maintain adequate oxygenation. Leave these patients alone! Closely monitor the patient, encourage him or her to continue to cough, offer supplemental oxygen, and transport to the hospital. Efforts to treat a mild airway obstruction with subdiaphragmatic (abdominal) thrusts may convert a mild airway obstruction to a severe one. Obviously, laryngoscopy is contraindicated in any conscious patient.

Treatment for a patient with an acute asthma attack should focus on which of the following goals? A) Rehydration and oxygenation B) Determination and correction of the cause C) Prompt transport to the nearest hospital D) Relief of the bronchospasm and improved ventilation

D Patients with asthma have two problems: bronchoconstriction and hypoxia; therefore, the goals of management are to relieve the bronchospasm and improve ventilation and oxygenation. Dehydration tends to occur in patients with asthma because of mucous plug formation and drying of the lower airway (children especially); therefore, fluid hydration may be required.

A patient presents with an acute onset of dyspnea. Which of the following conditions would be the LEAST likely underlying cause? A) Hyperventilation B) Pulmonary embolism C) Pneumothorax D) Pneumonia

D Pneumonia, an intrapulmonary infection, typically presents with a productive cough, fever and chills, and respiratory distress that gradually worsens. Hyperventilation, pulmonary embolism, and pneumothorax are all acute events, and thus present with acute dyspnea.

In which of the following situations would an endotracheal (ET) tube be of LEAST benefit to your patient? A) Cardiac arrest B) Unresponsiveness C) Prolonged ventilation D) Medication administration

D Research and evidence indicates that the endotracheal tube is not a reliable route for medication administration as was once thought. In fact, the onset of action of certain lipid-soluble drugs that can be given endotracheally (ie, lidocaine, epinephrine, atropine, naloxone) is unpredictable at best. Furthermore, the optimum endotracheal dose of those same drugs is unknown. Endotracheal intubation is indicated and is beneficial for patients in cardiac arrest, those who are unresponsive and unable to protect their own airway, or when prolonged positive-pressure ventilation is required.

Which of the following is MOST consistent with a patient who is hypoventilating? A) pH of 7.49 B) PO2 of 92 mm Hg C) Rapid, thready pulse D) PCO2 of 52 mm Hg

D Respiratory acidosis is always caused by hypoventilation. A patient who is hypoventilating is retaining carbon dioxide and is not bringing in enough oxygen. Therefore, the PCO2 increases (> 45 mm Hg) and the PO2 decreases (< 80 mm Hg). If the body cannot intake adequate amounts of oxygen, tissue oxygenation will ultimately suffer. As a result, the cells convert to anaerobic metabolism, which produces lactic acid and drives the pH down (< 7.35). A pH greater than 7.45 is consistent with metabolic alkalosis (ie, large quantities of ingested antacid) or respiratory alkalosis (ie, hyperventilation). Tachycardia may be present with both hypo- and hyperventilation.

Immediate treatment for an unresponsive patient with sonorous breathing involves: A) orotracheal intubation. B) inserting an oral airway. C) oropharyngeal suctioning. D) manual head positioning.

D Sonorous (snoring) breathing indicates partial blockage of the airway by the tongue. This is the most common cause of anatomic upper airway obstruction in patients with a decreased level of consciousness. The quickest way to correct this is to manually position the head (ie, head tilt-chin lift, jaw-thrust). Then, an oral or nasal airway should be inserted, which, in conjunction with manual head positioning, helps maintain patency of the airway. Intubation is indicated for patients who are unresponsive and unable to maintain their own airway; however, it should not precede basic airway maneuvers (eg, manual head positioning, basic airway adjunct) and preoxygenation with a bag-mask device. If secretions are jeopardizing a patient's airway, as evidenced by gurgling respirations, the oropharynx must be suctioned.

Which of the following statements regarding the concentration of gases is correct? A) The partial pressure of alveolar oxygen is about 75 torr. B) Atmospheric air contains approximately 25% to 30% oxygen. C) Fifteen percent of the gas within the alveoli is carbon dioxide. D) Nitrogen accounts for approximately 79% of atmospheric air.

D The majority of atmospheric gas is composed of nitrogen - approximately 79%. The partial pressure of oxygen in the alveoli is approximately 104 torr (mm Hg). Atmospheric (room) air contains 20.8% oxygen. The percentage of alveolar carbon dioxide is approximately 5%.

A 40-year-old male presents with acute respiratory distress while eating a meal. He is conscious and alert; is able to speak, but with difficulty; and has pink, moist skin. Which of the following statements BEST describes this patient's condition? A) Diffuse bronchospasm with good air exchange B) Severe airway obstruction with poor air exchange C) Progressive swelling of the upper airway structures D) Mild airway obstruction with adequate air exchange

D The patient in this scenario has a mild (partial) foreign body airway obstruction and is exchanging air adequately. He is conscious and alert; able to talk, although with difficulty; and is not cyanotic. Patients with a severe (complete) airway obstruction are not able to cough, talk, or breathe. They quickly become hypoxic, which manifests with a decreased level of consciousness and cyanosis. Given the circumstances in which the patient's acute difficulty breathing began - and the fact that he is not wheezing - bronchospasm is unlikely. Progressive upper airway swelling would be just that - progressive; this patient's difficulty breathing began suddenly. Furthermore, upper airway swelling would produce inspiratory stridor, which he does not have.

An unresponsive man is brought to the emergency department by his wife. Initial arterial blood gas analysis reveals a pH of 7.1, a PaO2 of 68 mm Hg, and a PaCO2 of 60 mm Hg. These findings are MOST consistent with: A) metabolic acidosis. B) metabolic alkalosis. C) respiratory alkalosis. D) respiratory acidosis.

D The patient is experiencing respiratory acidosis. A pH of less than 7.35 indicates acidosis; alkalosis of any kind is quickly ruled out by simply noting the low pH. An elevated (greater than 45 mm Hg) PaC02 indicates carbon dioxide retention, and a low (less than 80 mm Hg) PaO2 indicates hypoxia. This blood gas report is consistent with a patient who is hypoventilating.

Tidal volume is defined as the: A) volume of air moved in and out of the lungs each minute. B) maximum volume of air that the lungs can accommodate. C) residual volume of air in the lungs at the end of exhalation. D) volume of air moved in and out of the lungs per breath.

D Tidal volume is defined as the volume of air (in mL) that is moved in and out of the respiratory tract in a single breath. The volume of air moved in and out of the respiratory tract each minute is called minute volume, and is measured in liters. The average tidal volume of an adult male is approximately 500 mL. Of this amount, approximately 150 mL (1 mL per pound of body weight) remains in the anatomic dead space (trachea, large bronchi) and never reaches the alveolar level. Therefore, of the 500 mL of air that a 150-pound patient inhales, 350 mL actually reaches the alveoli (alveolar volume) and participates in pulmonary respiration; the remaining 150 mL lingers in the anatomic dead space until it is exhaled.

Ventilation of an adult patient with a stoma and no tracheostomy tube is MOST effectively achieved by: A) hyperextending the patient's head and ventilating with a pocket face mask. B) using a child-size bag-mask device and sealing the patient's mouth and nose. C) ventilating with a manually triggered ventilator attached to an adult-size mask. D) using an infant- or child-size mask attached to an adult-size bag-mask device.

D Ventilation of the stoma patient does not require manual head positioning (eg, head tilt-chin lift, jaw thrust); it can be performed with the patient's head in a neutral position. If the patient has a stoma and no tracheostomy tube, ventilations can be performed using the mouth-to-stoma (with a resuscitation mask) technique or with a bag-mask device. Regardless of the technique used, you should use an infant- or child-size mask to facilitate an adequate seal over the stoma. Using an infant- or child-size bag-mask device to ventilate an adult patient with a stoma would likely not provide adequate tidal volume.

With regard to endotracheal intubation, which of the following is the MOST harmful to your patient if unrecognized? A) Barotrauma B) Oral hemorrhage C) Tracheal necrosis D) Esophageal intubation

D When inserting an endotracheal (ET) tube, nothing will kill your patient quicker than inadvertently intubating the esophagus and not recognizing it! Not only must you visualize the ET tube passing between the vocal cords, you must also use additional methods of confirmation to ensure that the ET tube is in the trachea (ie, auscultation of breath sounds, quantitative waveform capnography). Barotrauma, oral hemorrhage, and tracheal necrosis are potential complications associated with endotracheal intubation and can cause harm to your patient. However, failure to ventilate the patient because the ET tube is in the esophagus is clearly the most lethal.

Which of the following MOST accurately describes the process of gas exchange in the lungs? A) The transfer of carbon dioxide from the alveoli into the bloodstream is facilitated by a process called diffusion. B) The gases exchanged in the lungs move from an area of greater concentration to an area of lesser concentration. C) Blood that returns to the lungs from the right side of the heart has a slightly lower level of carbon dioxide than oxygen. D) The partial pressure of oxygen in the alveoli is typically between 40 and 50 torr at the end of a maximal inhalation.

Pulmonary (external) respiration is defined as the exchange of gases in the lungs. Gases exchanged in the lungs (O2 and CO2) move from an area of greater concentration to an area of lesser concentration by a process called diffusion. Blood that enters the lungs from the right side of the heart has a PO2 of approximately 40 mm Hg and a PCO2 of approximately 46 mm Hg. Within the lungs, carbon dioxide diffuses from the bloodstream into the alveoli while oxygen diffuses from the alveoli into the bloodstream. The partial pressure of oxygen within the alveoli is near 100 mm Hg, while the partial pressure of carbon dioxide is near 0 mm Hg.


संबंधित स्टडी सेट्स

U2 What is the relationship between education and values?

View Set

Sadlier-Oxford Vocabulary Workshop Level G Units 9-10

View Set

Chapter 7 Policy Issuance and Delivery Exam

View Set

Lighting Distribution, Devices and Lighting Fixtures, Appliance Installation and Swimming Pools

View Set

Interactions Between Cells and Their Environment

View Set

ISTQB CTFL-Agile Tester ISTQB Cert Exam Example - OFFICIAL Exam

View Set